Gerald has [tex][tex][tex]$42 to buy fish at a pet store, and each fish costs $[/tex][/tex][/tex]4, including tax. Let m(x) represent the amount of money, in dollars, Gerald will have left after he
buys x fish. Which choice is the most appropriate domain for function m?
A (1, 3, 5, 7, 9, 11)
B (0, 1, 2, 3, 4, 5, 6, 7, 8, 9, 10, 11)
(0, 2, 4, 6, 8, 10)
(0, 1, 2, 3, 4, 5, 6, 7, 8, 9, 10)



Answer :

Other Questions